Derivation of the Eqation of Motion from Fermi Lagrangian density

In summary, The discussion is about finding the equations of motion derived from the Fermi Lagrangian density in the covariant formalism of Electromagnetism. The Lagrangian density is given by $L=-\frac{1}{2} (\partial_n A_m)(\partial^n A^m) - \frac{1}{c} J_m A^m$, where J is the electric current. The desired result is $\partial_n \partial^n A^m = \frac{1}{c} J^m$. The Euler-Lagrange equations are used to derive this result, but there is confusion about how to treat the fields $A_m$ and $A^m$. The OP also mentions a different
  • #1
radioactive8
46
0

Homework Statement


Hello, I am trying to find the equations of motion that come from the fermi lagrangian density of the covariant formalism of Electeomagnetism.

Homework Equations


The form of the L. density is:
$$L=-\frac{1}{2} (\partial_n A_m)(\partial^n A^m) - \frac{1}{c} J_m A^m$$

where J is the electric current.
The result has to be:

$$\partial_n \partial^n A^m = \frac{1}{c} J^m$$

The Attempt at a Solution


Using the Euler- Lagrange equations that derive the eq. of motion I do not understant if I have to treat the fields Am and Am. In addition, the Euler Lagrange equations from a L.density of the form:
$$L=-\frac{1}{2} (\partial_n A_m)(\partial_n A_m) - \frac{1}{c} J_m A^m$$
giveaway the wanted result. But I could not relate the two L.densities.
 
Physics news on Phys.org
  • #2
How did you lower the indices to get the density in §3 ?
I think the density in §3 is not covariant and it is then incorrect.
There must be elements of the metric tensor missing in the density of §3 .
 
  • #3
maajdl said:
How did you lower the indices to get the density in §3 ?
I think the density in §3 is not covariant and it is then incorrect.
There must be elements of the metric tensor missing in the density of §3 .

Indeed. OP, please show your work.
 
  • #4
I did not lower any indices. And yes it is not covariant at all so it could not be a lagrangian. It was something a digged up on the internet while I was working at the solution of my problem. However, I did not notice while posting the thread that it was not covariant.

Now, Let's focus on the first L.density.

I tried playing with the indexes and my metric.

Is the following correct? :

$$L=-1/2(\partial_n A_m)( \partial^n A^m) = -\frac{1}{2} (g_{ns} \partial^s A_m)(g^{nr} \partial_r A^m) = -\frac{1}{2} \delta_{s}^{r}(\partial^s A_m)( \partial_r A^m)= -\frac{1}{2} (\partial^s A_m)(\partial_s A^m) $$

So the partial derivative $$\frac{ \partial L}{\partial (\partial_s A^m)}$$ equals:

$$\partial_s (\frac{ \partial L}{\partial (\partial_s A^m)})= \partial_s \partial^s A^m$$
 

1. What is the Fermi Lagrangian density?

The Fermi Lagrangian density is a mathematical expression used in quantum field theory to describe the dynamics of a system of interacting particles. It is derived from the Lagrangian function, which represents the total energy of a system as a function of its coordinates and velocities.

2. How is the equation of motion derived from the Fermi Lagrangian density?

The equation of motion can be derived from the Fermi Lagrangian density by applying the principle of least action, which states that the path taken by a system between two points in time is the one that minimizes the action (a measure of the system's energy) along that path. This leads to the Euler-Lagrange equations, which describe the motion of particles in the system.

3. What is the significance of the equation of motion in physics?

The equation of motion is a fundamental concept in classical and quantum mechanics, as it describes the behavior of particles and systems in motion. It allows us to predict the future positions and velocities of particles based on their initial conditions and the forces acting on them.

4. Can the equation of motion be applied to all types of systems?

The equation of motion derived from the Fermi Lagrangian density is applicable to all systems that can be described using quantum field theory, such as elementary particles and their interactions. However, it may not be applicable to systems that involve relativistic effects or strong gravitational fields, in which case other equations of motion may need to be used.

5. Are there any alternative methods for deriving the equation of motion from the Fermi Lagrangian density?

Yes, there are other mathematical approaches that can be used to derive the equation of motion from the Fermi Lagrangian density, such as the Hamiltonian formalism or the path integral formulation. These methods may provide different insights and perspectives on the dynamics of the system.

Similar threads

  • Advanced Physics Homework Help
Replies
10
Views
1K
  • Advanced Physics Homework Help
Replies
1
Views
1K
  • Advanced Physics Homework Help
Replies
5
Views
1K
  • Advanced Physics Homework Help
Replies
2
Views
827
Replies
5
Views
2K
  • Advanced Physics Homework Help
Replies
0
Views
665
  • Advanced Physics Homework Help
Replies
18
Views
2K
  • Advanced Physics Homework Help
Replies
2
Views
2K
  • Advanced Physics Homework Help
Replies
0
Views
135
  • Advanced Physics Homework Help
Replies
16
Views
915
Back
Top